ChaseDream

标题: GWD-11-Q38 [打印本页]

作者: apriltong    时间: 2010-4-20 01:24
标题: GWD-11-Q38
Unprecedented industrial growth in the country of Remo has created serious environmental problems because factories there lack adequate pollution-control systems.  Remo is developing a clean growth plan that includes environmental regulations that will require the installation of such systems.  Since no companies in Remo currently produce pollution-control systems, the plan, if implemented, will create significant opportunities for foreign exporters to market pollution-control systems.
Which of the following, if true, most seriously weakens the argument?
A.    The clean growth plan will provide tax incentives for local businesses to develop and manufacture pollution-control devices.
B.    Foreign exporters would provide factory-trained technicians to maintain the pollution-control systems sold to Remo.
C.    Industrial lobbyists sponsored by local businesses in Remo are trying to prevent the implementation of the government regulations.
D.    The regulations that Remo plans to implement are much less strict than those in neighboring nations.
E.    ollution in Remo has caused serious health problems for workers, contributing to a significant increase in the number of workdays lost to illness.


正确答案是A,这个我明白了,因为国内还没有人造这个,所以国外的出口商就可以赚钱了。假设是国内以后也没人造,A削弱了假设。
但是有点纠结于C,开始游说政府取消这个政策,那国外出口商不是更加没有钱赚了吗?
请大家指点
作者: guanghenry    时间: 2010-4-20 10:57
C不可以,因为削弱的必须是结论或是从前提到结论的过程而非前提,C就是削弱了前提,不符合要求!
作者: apriltong    时间: 2010-4-20 15:24
哦,明白了,因为argument里面已经说了 if the plan implenmented..., 所以结论是建立在这个基础上的。
谢谢指点
作者: shalinzz    时间: 2010-5-31 23:33
我选的D,我上来就排除了A,感觉它无关,没紧扣export
这题感觉哪个选项都不好!




欢迎光临 ChaseDream (https://forum.chasedream.com/) Powered by Discuz! X3.3